Doomsday Mathematical Olympiad: 2 Set of Sample Problems

Download as pdf or txt
Download as pdf or txt
You are on page 1of 13
At a glance
Powered by AI
The passage proves that (2k - 1)!! divides the difference between binomial coefficients using combinatorial and number theoretic arguments.

The proof uses modular arithmetic and properties of permutations and sums to show congruences.

P1, P2 and P3 are defined as products involving terms of the form (2k + i) where i ranges from 1 to 2k. P1, P2 and P3 are used to write the difference between binomial coefficients as a combination of other terms.

Doomsday Mathematical Olympiad

2nd Set of sample problems

Dooms Mathematical Association


Samples are Not original problems
DMO Sample Problems

1. Let a, b, c be positive numbers such that abc= 1. Prove that


1 1 1 3
+ + ≥
a3 (b + c) b3 (c + a) c3 (a + b) 2
(beware its very popular)

Solution
After the setting a = x1 , b = y1 , c = z1 , and as abc = 1 so a1 . 1b . 1c = 1 concluding
xyz = 1.
x2 y2 z2 3
Claim : + + ≥
y+z z+x x+y 2
Now By Cauchy Schwarz Inequality,
 2
y2 z2

x
[(y + z) + (z + x) + (x + y)] + + ≥ (x + y + z)2
y+z z+x x+y
By Titu Lemma .I guess all know what titu lemma is,its a cauchy schwarz
Inequality in engel’s form,
x2 y2 z2 (x + y + z)2
=⇒ + + ≥
y+z z+x x+y 2(x + y + z)
x2 y2 z2 (x + y + z)
=⇒ + + ≥
y+z z+x x+y 2
Now by AM-GM we know that

(x + y + z) ≥ 3 3 xyz

and xyz = 1 which concludes to =⇒ (x + y + z) ≥ 3 3 1 Therefore we get
x2 y2 z2 3
=⇒ + + ≥
y+z z+x x+y 2
Hence our claim is proved

1
Doomsday Mathematical Olympiad Team DMO

2. Let n be a positive integer with n ≥ 2. Fix 2n points in space in such a way


that no four of them are in the same plane, and select any n2 + 1 segments
determined by the given points. Prove that these segments form at least one
triangle.

Solution
We use induction
 on n. If n = 2, then 5segments
 are drawn. There are at
4 4
most = 6 segments and they form = 4 triangles. Note that each
2 3
segment is part of two triangles. The removal of one of the 6 segments will
break up exactly two of the 4 triangles. Hence there are 2 triangles formed
by the 5 segments. We assume that the statement in the problem is true
for n = 1, 2, . . . , k, where k is a positive integer. Now suppose we are given
2n = 2(k + 1) points P1 , P2 , . . . , P2k+2 , and that (k + 1)2 + 1 segments are
constructed. We know that three of these segments form a triangle. Without
loss of generality, assume that the triangle is P1 P2 P3 . We say that points
Pi and Pj (1 ≤ i < j ≤ n) are connected if Pi Pj is one of the constructed
segments. Let S = {P4 , . . . , P2k+2 } , and for i = 1, 2, 3, let Si denote the
elements in S that are connected to Pi . If Pk is in Si ∩ Sj , then Pi Pj Pk is
a triangle. Then the segments form at least tk+1 = |S1 ∩ S2 | + |S2 ∩ S3 | +
|S3 ∩ S1 | triangles. By the Inclusion-Exclusion Principle, we have

|S1 ∪ S2 ∪ S3 | = |S1 | + |S2 | + |S3 | − tk+1 + |S1 ∩ S2 ∩ S3 |

implying that
tk+1 ≥ |S1 | + |S2 | + |S3 | − (2k − 1)
because |S1 ∪ S2 ∪ S3 | ≤ 2k + 2 − 3 = 2k − 1 and |S1 ∩ S2 ∩ S3 | ≥ 0

If |S1 |+|S2 |+|S3 | ≥ 3k−1, then tk+1 ≥ k; that is, there are at least k triangles
besides triangle P1 P2 P3 , and we are done. We assume that |S1 | + |S2 | + |S3 | ≤
3k − 1, or

(|S1 | + |S2 |) + (|S2 | + |S3 |) + (|S3 | + |S1 |) ≤ 6k − 2

Hence, by the Pigeonhole Principle, at least one of |S1 | + |S2 | |S1 | + |S2 | , and
|S1 | + |S2 | is no greater than the average 6k−2
3 Without loss of generality, we
may assume that |S1 | + |S2 | ≤ 2k − 1 (because |S1 | + |S2 | is an integer). If we
remove P1 and P2 , we lose at most |S1 | + |S2 | + 1 = 2k segments. Therefore,
there are at least (k + 1)2 − 2k = k 2 + 1 segments constructed among the 2k

2
Doomsday Mathematical Olympiad Team DMO

points P3 , P4 , . . . , P2k+2 . By the induction hypothesis, the segments form at


least k triangles. Thus we have at least k+1 triangles in the given configuration
of points P1 , P2 , . . . , P2k+2 , and our induction is complete.

3. Do there exist sequences of positive real numbers in which each term is the
product of the two previous terms and for which all odd-numbered terms are
greater than 1, while all even-numbered terms are less than 1? If so, find all
such sequences. If not, prove that no such sequence is possible.

Solution:Let (an )n≥0 be such a sequence of positive numbers, and let bn =


ln an for each n. Then we have bn+2 = bn+1 + bn for n ≥ 0, and the terms
alternate in sign, with b0 > 0. By the standard method for solving linear
recurrence relations with constant coefficients, we find
√ !n √ !n
1+ 5 1− 5
b n = c1 + c2
2 2
√ √
1+ 5 1− 5
for some constants c1 and c2 . since 2 > 1 and −1 < 2 < 0, the terms
will alternate in the prescribed manner if and only if c1 =√ 0 and c2 >√0. This
means that a = a0 = eb0 = ec2 > 1, and a1 = eb1 = ec2 (1− 5)/2 = a(1− 5)/2

4. Each of the vertices of a regular nonagon has been colored either red or blue.
Prove that there exist two congruent monochromatic triangles; that is, trian-
gles whose vertices are all the same color.

We call a monochromatic triangle red (blue) if all of its vertices are red (blue).
Because there are nine vertices colored in two colors, at least five must be of
the same color. Without  loss
 of generality, we say that this color is red.
5
Hence there are at least = 10 red triangles. We now prove that there
3
are two congruent red triangles. Let A1 , A2 , · · · , A9 denote the vertices of
the nonagon (Figure ), and let ω be the its circumcircle. The vertices of the
nonagon cut ω into nine equal arcs. We call each of these nine arcs a piece.
Let Ai Aj Ak be a triangle with Ai Aj ≤ Aj Ak ≤ Ak Ai . Denote by ai,j the
number of pieces in the arc Ai Aj , not containing the point Ak and define

3
Doomsday Mathematical Olympiad Team DMO

aj,k and ak,i analogously. Then define a map that maps the triangle Ai Aj Ak
to the triple (ai,j , aj,k , ak,i ) . It is clear that 1 ≤ ai,j ≤ aj,k ≤ ak,i ≤ 7 and
ai,j + aj,k + ak,i = 9. For example, the triangle with vertices A2 , A4 , A8 is read
as triangle A4 A2 A8 and mapped to the triple (2,3,4) for the diagram visit here
https://artofproblemsolving.com/community/c1156956h2152349_combi_geometry
_from_andresscu Congruent triangles map to the same triple, while incongru-
ent triangles map to distinct triples. Hence we have built a bijection between
the classes of congruent triangles and the set of ordered triples of positive inte-
gers (a, b, c) with a ≤ b ≤ c and a + b + c = 9 It is not difficult to list all such
triples: (1,1,7),(1,2,6),(1,3,5) (1, 4, 4), (2, 2, 5), (2, 3, 4), (3, 3, 3). Hence there
are seven classes of congruent triangles. since there are at least 10 red trian-
gles, some class must contain at least two red triangles and hence there are at
least two congruent red triangles.

5. In a scalene triangle ABC, the incircle ω has center I and touches side BC at
D. A circle Ω passes through B and C and intersects ω at two distinct points.
The common tangents to ω and Ω intersect at T , and line AT intersects Ω
at two distinct points K and L. Prove that either KI bisects ∠AKD or LI
bisects ∠ALD.

We begin with the following lemma. Lemma. A circle Γ passes through B


and C in 4ABC; point M is the point on Γ lying on the same side of BC
−−→
as A, such that M B = M C. Let I be the incenter of 4ABC. Ray M I
meets Γ again at X. Then a circle ω exists, passing through X, tangent to Γ,
AB, and AC. In particular, invert at X and proceed similarly.  Let M, N
be the midpoints of arc BCd in Ω, with M and A on the same side of BC.
Let O be the center of Ω and L = KM ∩ RT . Construct a circle γ passing
through K, touching sides AB and AC. By Monge D’Alembert’s theorem,
the exsimilicenters of Ω, ω, γ are collinear; so since K lies on AT we see that
γ, Ω touch at K. The lemma shows that K, I, M are collinear. Note that T is
the center of positive homothety, such that ω 7→ Ω, hence T, I, O and T, R, N
are collinear. Observe that
K I
(KA, KR; KM, KN ) = (T R; LN ) = (O∞, M N ) = −1
hence KI bisects angle T KR, as desired.  2nd Solution Let M denote the
midpoint of the arc BC of Ω which is on the same side of BC as A. We claim

4
Doomsday Mathematical Olympiad Team DMO

that if we let P = M I ∩ Ω ∈ {K, L}, then P I bisects ∠AP D and P ∈ AT. In


fact, we will make the even stronger claim that P I bisects both ∠BP C and
∠AP D. It is immediate that P I bisects ∠BP C since P I goes through M.
The following claim then implies that P I bisects ∠AP D as well: Claim. The
set of points for which ∠AP I = ∠IP D is exactly the same as that for which
∠BP I = ∠IP C. (Note: Here, angles are directed) Proof . Let’s call the first
set (∠AP I = ∠IP D) S1 and the second set S2 . Let’s invert the entire figure
with respect to I. Our claim is now equivalent to the following new problem:
New Problem. Let 4D0 E 0 F 0 be a triangle with circumcenter I, and let the
midpoints of its sides be A0 ∈ E 0 F 0 , B 0 ∈ D0 F 0 , and C 0 ∈ D0 E 0 . Show that
the set of points P for which ∠IB 0 P = ∠P C 0 I is the same as the set of
points P for which ∠IA0 P = ∠P D0 I. As usual, angles directed here. Now,
we’ll show this new problem. Let lowercase letters correspond to the complex
coordinates of any point. Then, observe that the condition is equivalent to
(p − b0 )(p − c0 ) ∈ (i − b0 )(i − c0 )R, where we note that (i − b0 )(i − c0 ) is a
0
)(p−c0 )
constant. Hence, equating (p−b (i−b )(i−c0 ) with its conjugate and letting p = x + yi
0

gives a degree 2 equation in x and y, i.e., a conic. Analogously, S2 is also


a conic, so it suffices to verify that S1 and S2 coincide in at least 5 points.
Firstly, it’s trivial to verify that A0 , B 0 , C 0 , D0 are four points which work.
Now, let H 0 be the orthocenter of 4D0 B 0 C 0 . Then, notice that H 0 A0 ID0 and
H 0 B 0 IC 0 are both parallelograms, and so hence H 0 is also on both S1 and
S2 . As a result, S1 ≡ S2 since we’ve found five points lying on both conics.
Now invert back and we’re done.  The following claim will prove critical
in showing P ∈ AT. Claim. Let’s draw a circle Γ which is tangent to Ω
internally at a point Q on the opposite of BC as A, to AB, and to BC.
Then Q = P. Proof . Let AB, AC touch the circle Γ at U, V, respectively.
Let B 0 , C 0 be the points where AB, AC meet Ω for a second time. Let X, Y
denote the incenters of BCB 0 , BCC 0 , respectively. Then, it’s well-known by
Sawayama-Thebault that X, Y, U, V are all collinear. Note that ∠Y U B =
∠V U B = 90 + ∠A 2 = ∠BIY. Therefore, BU IY is cyclic. By homothety, it’s
well-known that ∠BQU = 12 ∠BCB 0 . Also, ∠BY U = ∠AU Y − ∠Y BA =
90 − 12 ∠A − 12 ∠B − 12 ∠ABC 0 = 12 ∠C − 21 ∠ABC 0 = 12 ∠BCB 0 , and so hence
BU Y Q is also cyclic. As a result of the previous two cyclic quadrilaterals, we
know that BU IY Q is cyclic. Analogously, so is CV IXQ. Hence, it follows
that ∠BQI = ∠IU A = ∠IV A = ∠IQC, where we used symmetry in the
middle step (AU = AV and I being on the angle bisector). This clearly
implies what we want.  By Monge’s theorem Ton Γ, ω, Ω, we know that

5
Doomsday Mathematical Olympiad Team DMO

A, T, Q = P are collinear. Therefore, it follows that P ∈ AT. We’ve also


shown that P I bisects ∠BP C and ∠AP D and so hence we are done.

6. Find all non-decreasing functions f : Z → Z which satisfy

f (k) + f (k + 1) + . . . + f (k + n − 1) = k

for any k ∈ Z, and fixed n

Solution:Again subtracting the condition for k from the condition for k +


1 we get f (k + n) = f (k) + 1. Therefore f is determined by its’ values
on {0, 1, . . . , n − 1} and the relation f (k) = nk + f (kmodn) As f is non-
decreasing and f (n) = f (0) + 1, we see that there is a

0 ≤ m ≤ n − 1 such that f (0) = f (1) = . . . = f (m), f (0) + 1 =

f (m+1) = . . . = f (n). Now by writing the condition for k = 0 we get nf (0)+


(n−m−1) = 0 which impliesm = n−1 thus f (0) = f (1) = . . . = f (n−1) =
0. It’s now clear that f (k) = nk . This value clearlysatisfies the condition, as
1 n−1

it is a consequence of Hermite’s Identity [x]+ x + n +. . .+ x + n = [nx].
Note that we have also proven the identity by induction during the proof.

7. An ordered pair (x, y) of integers is a primitive point if the greatest common


divisor of x and y is 1 . Given a finite set S of primitive points, prove That
there exist a positive integer n and integers a0 , a1 , . . . , an such that, for each
(x, y) in S, we have:

a0 xn + a1 xn−1 y + a2 xn−2 y 2 + · · · + an−1 xy n−1 + an y n = 1

Sol:We prove the result by induction on |S|, with the base case being Be-
zout’s Lemma (n = 1). For the inductive step, suppose we want to add a
given pair (am+1 , bm+1 ) to {(a1 , . . . , am ), (b1 , . . . , bm )}. By a suitable linear
transformation assume (am+1 , bm+1 ) = (1, 0). (The transformation is not nec-
essary to proceed but cleans up the presentation that follows.) Let g(x, y) be

6
Doomsday Mathematical Olympiad Team DMO

a polynomial which works on the latter set. We claim we can choose the new
polynomial f of the form
m
Y
M deg g·M −m
f (x, y) = g(x, y) − Cx (bi x − ai y).
i=1

where C and M are integer parameters we may adjust. Since f (ai , bi ) = 1 by


construction we just need
Y
M
1 = f (1, 0) = g(1, 0) − C bi .
Q
If bi = 0 we are done, since bi = 0 =⇒ ai = ±1 in that case and so
g(1, 0) = ±1, thus take M = 2. So it suffices to prove: Claim: gcd (g(1, 0), bi ) =
1 when bi 6= 0. Proof. Fix i. If bi = 0 then ai = ±1 and g(±1, 0) = ±1.
Otherwise know
1 = g(ai , bi ) ≡ g(ai , 0) (mod bi )
and since the polynomial is homogeneous with gcd(ai , bi ) = 1 it followsQ
g(1, 0) 6≡ 0 (mod bi ) as well.  Then take M a large even multiple of ϕ( bi )
and we’re done

8. Any positive integer m can be written uniquely in base 3 form as a string of


00 s, 1 ’s and 2 ’s (not beginning with a zero). For example

98 = 81 + 9 + 2 × 3 + 2 × 1 = (10122)3

Let c(m) denote the sum of the cubes of the digits of the base 3 form of m;
thus, for instance

c(98) = 13 + 03 + 13 + 23 + 23 = 18

Let n be any fixed positive integer. Define the sequence {ur } as

u1 = n, and ur = c (ur−1 ) for r ≥ 2

Show that there is a positive integer r such that ur = 1, 2, or 17

SOLUTION:If m has d ≥ 5 digits then we have m ≥ 3d−1 = (80+ 1) (d−1)/4 ≥


80 · d−1
4 + 1 > 8d by Bernoulli’s inequality. Thus m > c(m) If m > 32
has 4 digits in bese 3, then c(m) ≤ 23 + 33 + 23 + 23 = 32 < m On the

7
Doomsday Mathematical Olympiad Team DMO

other hand, if 27 ≤ m ≤ 32, then m starts with the digits 10 in base 3 and
c(m) < 13 + 03 + 23 + 23 = 17 < m Therefore 0 < c(m) < m for all m ≥ 27.
Hence, eventually, we have ua < 27. Because us hus at most three digits,
us+1 can only equal 8, 16, 24, 1, 9, 17, 2, 10, or 3. If it oquals 1, 2, or 17 we are
already done; if it equals 3 or 9 then us+2 = 1. Otherwise a simple check shows
that ur will eventually will be 2,

8 = (22)3
→ 16 = (121)3 → 10 = (101)3 → 2
24 = (220)3

9. We are given triangles ABC and DEF such that D ∈ BC, E ∈ CA, F ∈
AB, AD ⊥ EF, BE ⊥ F D, CF ⊥ DE. Let the circumcenter of DEF be
O, and let the circumcircle of DEF intersect BC, CA, AB again at R, S, T
respectively. Prove that the perpendiculars to BC, CA, AB through D, E, F
respectively intersect at a point X, and the lines AR, BS, CT intersect at a
point Y , such that O, X, Y are collinear.

SOLUTION:We know that AD, BE, CF concur at the orthocenter of DEF ,


denoted by H. Let X 0 denote the isogonal conjugate of H wrt ABC, If
DEF isn’t the pedal triangle of X 0 , then its pedal triangle is homothetic
with DEF which is impossible since the lines that connect the corresponding
points of the two homothetic triangles don’t concur. Hence X 0 ≡ X. It’s
a well known property of isogonal conjugates that O, X, H are collinear and
RST is the pedal triangle of H. We will prove that BS ∩ CT = Y 0 lies
on line OXH, in which it’s clear that the concurrence at Y follows. Apply
Pappus’ theorem on CES, BF T and we have H, Y 0 , ET ∩ F S are collinear.
Let HS, HT ∩(O) = S 0 , T 0 again and since SS 0 ∩T T 0 = O, by Pascal theorem
we have O, H, ET ∩ F S are collinear and hence O, X, H, ET ∩ F S, Y 0 are
collinear and we are done.

10. For every integer k ≥ 2, prove that 23k divides the number
 k+1   k 
2 2

2k 2k−1
but 23k+1 does not.

8
Doomsday Mathematical Olympiad Team DMO

SOLUTION:(For odd integers n, (2n − 1)!! = 1 · 3 · · · (2n − 1).) Lemma 1:


v2 ((2n )!) = 2n − 1. Proof:
 n  n  n
2 2 2
v2 ((2n )!) = 1 + 2 + · · · + n
2 2 2
n−1 n−2
=2 +2 + ... + 1
= 2n − 1.

Corollary 1: v2 ((2k−1 + 1)(2k−1 + 2) · · · (2k−1 + 2k−1 )) = 2k−1 . Proof:


 k 
(2 )!
v2 ((2k−1 + 1)(2k−1 + 2) · · · (2k−1 + 2k−1 )) = v2
(2k−1 )!
= (2k − 1) − (2k−1 − 1)
= 2k−1 .
k−1 k−1 k−1 k−1
Lemma 2: (2 +1)(2 2+2)···(2
2k−1
+2 )
= (2k − 1)!!. Proof: For every positive
odd integer i less than 2k , there exists a unique integer bi such that 2k−1 <
i2bi < 2k . Conversely, for every integer c between 2k−1 and 2k , there exists a
unique odd integer i such that i2bi = c. Furthermore, b1 +b3 +. . .+b2k −1 must
be equal to the exponent of the greatest power of 2 dividing (2k−1 + 1)(2k−1 +
2) · · · (2k−1 + 2k−1 ), which by corollary 1 equals 2k−1 . It follows that

(2k−1 + 1)(2k−1 + 2) · · · (2k−1 + 2k−1 ) = (1 · 2b1 )(3 · 2b3 ) · · · (2k−1 · 2b2k −1 )


= (2k − 1)!!2b1 +b2 +...+b2k −1
k−1
= (2k − 1)!!22

which completes our proof. Let P1 = (2k + 1)(2k + 2) · · · (2k + 2k ), let P2 =


(2k−1 + 1)(2k−1 + 2) · · · (2k−1 + 2k−1 ), and let P3 = (2k + 1)(2k + 3) · · · (2k +
2k − 1). Note that

P1 = (2k + 1)(2k + 2) . . . (2k + 2k )


= (2k + 1)(2k + 3) . . . (2k + 2k − 1) 2(2k−1 + 1) · 2(2k−1 + 2) · · · 2(2k−1 + 2k−1 )
 
k−1
= 22 P 2 P3

9
Doomsday Mathematical Olympiad Team DMO

Now,
 k+1   k 
2 2 (2k+1 )! (2k )!
− k−1 = k 2 − k−1 2
2k 2 (2 )! (2 )!
(2k+1 )! (2k )!((2k−1 + 1)(2k−1 + 2) · · · (2k−1 + 2k−1 ))2
= k 2 −
(2 )! (2k )!2
(2k )!((2k + 1)(2k + 2) · · · (2k + 2k )) − (2k )!((2k−1 + 1)(2k−1 + 2) ·
=
(2k )!2
P1 (2k )! − (2k )!P22
=
(2k )!2
P1 − P22
=
(2k )!
k−1
22 P2 P3 − P22
=
(2k )!
k−1
22 P2
 
P2
= P3 − 2k−1
(2k )! 2
 2k−1 
By lemma 1 and corollary 1, v2 2 (2k )!P2 = 1. Also, by lemma 2, 22Pk−1 2
=
(2k − 1)!!, so we want to show that
23k−1 ||(2k + 1)(2k + 3) · · · (2k + 2k − 1) − (2k − 1)!!.
Let S1 = 11 + 31 + . . . + 2k1−1 , let S2 = 1≤i<j≤2k−1 (2i−1)(2j−1) 1 1 1
P
= 1·3 + 1·5 +· · ·+
1 1 1 1 1 1
1·(2k −1)
+ 3·5 + · · · + (2k −3)·(2k −1) , and let S3 = 1·(2k −1) + 3·(2k −3) + · · · + (2k −1)·1 .
Note that
2S1
S1 =
2
1 1 1 1 1 1
  
+ k + + 2k −3
+ . . . + 2k −1
+
= 1 2 −1 3 1
2
S3
= 2k
2
k−1
= 2 S3
We have that
P3 − (2k − 1)!! = (2k + 1)(2k + 3) · · · (2k + 2k − 1) − (2k − 1)!!
= 23k (some terms) + (2k − 1)!!(22k S2 + 2k−1 S1 )
= 23k (some terms) + (2k − 1)!!(22k S2 + 22k−1 S3 )

10
Doomsday Mathematical Olympiad Team DMO

We wish to show that 23k−1 ||23k (some terms) + (2k − 1)!!(22k S2 + 22k−1 S3 ),
i.e., 2k−1 ||S2 + S21 . This is clearly equivalent to S2 + S21 ≡ 2k−1 (mod 2k ).
Since { 11 , 13 , · · · , 2k1−1 } is a permutation of {1, 3, · · · , 2k − 1} modulo 2k and
2k − a ≡ −a (mod 2k ), we have:
S2 ≡ 1 · 3 + 1 · 5 + · · · + 1 · (2n − 1) + 3 · 5 + · · · + (2k − 3)(2k − 1)
(1 + 3 + · · · + 2k − 1)2 − (12 + 32 + · · · + (2k − 1)2 )
≡ (mod 2k ),
2
k 2
−1)
and S3 ≡ −(12 + 32 + · · · + (2k − 1)2 ) (mod 2k ). 2k |22k−1 = (1+3+···+2
2 , so
S3 2 3 n 2 k
S2 + 2 ≡ −(1 + 3 + · · · + (2 − 1) ) (mod 2 ). Hence,
 
S3
− S2 + ≡ 12 + 32 + · · · + (2k − 1)2
2
= 12 + 22 + · · · + (2k − 1)2 − 22 (12 + 22 + · · · + (2k−1 − 1)2 )
2k (2k − 1)(2k+1 − 1) 2k−1 (2k−1 − 1)(2k − 1)
= −4
6 6
k−1 k k+1 k−1 k−1
2 (2 − 1)(2 − 1) − 2 · 2 (2 − 1)(2k − 1)
=
 k+1 3
k−1

2 − 1 − 2(2 − 1)
= 2k−1 (2k − 1)
3
 k+1 k

2 − 1 − 2 + 2
= (2k − 1)(2k−1 )
3
 k 
2 +1
= (2k − 1)(2k−1 )
3
 k 
4 −1
= 2k−1
3
≡ 2k−1 (mod 2k )
which is congruent to 2k−1 modulo 2k , as desired.

11.FExtra Let n be a positive integer. Each point (x, y) in the plane, where x and y are
non-negative integers with x + y < n, is coloured red or blue, subject to the
following condition: if a point (x, y) is red, then so are all points (x0 , y 0 ) with
x0 ≤ x and y 0 ≤ y. Let A be the number of ways to choose n blue points
with distinct x-coordinates, and let B be the number of ways to choose n blue
points with distinct y-coordinates. Prove that A = B.

11
Doomsday Mathematical Olympiad Team DMO

FFFFFF Here is the end FFFFFF

Regards,
Team DMO

12

You might also like